A n+-1 Solutions - Yufei Zhao - Trinity Training 2011
A n+-1 Solutions - Yufei Zhao - Trinity Training 2011
A n+-1 Solutions - Yufei Zhao - Trinity Training 2011
Yufei Zhao
an 1
Solutions
Yufei Zhao
Trinity College, Cambridge
yufei.zhao@gmail.com
April 2011
Practice problems:
1. A primitive root mod n is a number g such that the smallest positive integer k for which
g k 1 mod n is (n).
(a) Show that 2 is a primitive root mod 3n for any n 1.
(b) Show that if g is an odd primitive root mod p such that p2 - g p1 1, then g is also
a primitive root mod pn and 2pn for any n 1.
n1
Solution. (a) Since (3n ) = 2 3n1 , the problem amounts to showing that 3n - 23
1
n2
and 3n - 223
1 (when n 2). The first claim follows from reduction mod 3, and the
n2
second claim follows from the exponent lifting trick, as 3 k 22 1, so that 3n1 k 223 1.
n2
d|n d (x)
an 1
Yufei Zhao
n
k
gcd(p1,n) 1.
gcd(p1,n) . Suppose p k m
mn 1. However, from (a) we know that
By
1
= pp1 + pp2 + + p + 1 with
Solution. Let q be a prime divisor of p (p) = pp1
p2 - q 1 (this must exist since p (p) 6 1 (mod p2 )). By problem 2, p | q 1. If np p
q1
q1
gcd( q1 ,p)
p
(mod q), then p p nq1 1 (mod q). We have q | gcd(p p 1, pp 1) = p
1,
2
which equals to p 1 since p - q 1. However, we cannot simultaneously have q | p 1
and p | q 1. Thus np p is not divisible by q.
4. (a) Prove that m (x) and n (x) are always relatively prime as polynomials for m 6= n.
(b) Show that if for some integer x, m (x) and n (x) are not relative prime, then m/n
is an integer power of a prime.
Solution. (a) The zeros of n (x) and m (x) are distinct, since the zeros of n (x) are
precisely the primitive n-th roots of unity. Thus the polynomials are relatively prime.
(b) Suppose some prime p divides both m (x) and n (x). By replacing x by x + p if
necessary, we may assume that x > 1. Let us deal with the p = 2 case separately. We
claim that if m (x) is even then m must be a power of 2. Indeed, otherwise let q be an
odd prime divisor of m, and let m = qs, then by the previous problem, m (x) divides
xm 1
(q1)s + x(q2)s + + xs + 1, which is always odd. The p = 2 case follows.
xs 1 = x
Now assume that p > 2. By the previous problem, p divides xm 1 and xn 1, and hence
m
n
p | xgcd(m,n) 1. Let pk k xgcd(m,n) 1. One of gcd(m,n)
and gcd(m,n)
is not divisible by
p, and assume that it is the latter. Then by the exponent lifting trick, pk k xn 1. If
xn 1
gcd(m, n) < n, then n (x) divides xgcd(m,n)
, which is not divisible by p by the above
1
analysis. This contradicts p | n (x). Hence gcd(m, n) = n, i.e., n | m.
m
We claim that m
n is a power of p. If not, then pick some prime q dividing n . We have
p | n (x) | xn 1 | xm/q 1. By the exponent lifting trick, the same power of p divides
m
both xm 1 and xm/q 1. But m (x) divides xxm/q1
, which contradicts p | m (x). Thus
1
m
n is a power of p.
Solution. (a) Observe that if a and b are coprime odd numbers, then gcd(2a + 1, 2b + 1) =
3, since their gcd must divide gcd(22a 1, 22b 1) = 2gcd(2a,2b) 1 = 22 1 = 3. Since
2ab + 1 is divisible by both 2a + 1 and 2b + 1, it must also be divisible by 31 (2a + 1)(2b + 1).
2
an 1
Yufei Zhao
Consider the set of divisors d of p1 pk with an odd number of prime factors. There are
2k1 such divisors d, and they provide mutually coprime d (2) by Problem 4. Take one
prime divisor from each such d (2) and we get what we want.
6. (IMO 1990) Determine all positive integers n such that
2n + 1
is an integer.
n2
an 1
Yufei Zhao
1 k m, the k-th digit from the right in n is odd if and only if the k-th digit from the
right in N is odd).
Solution. We will prove by induction on m that there exists infinitely many n that works.
This is trivial when m = 1, 2.
For the inductive step, it suffices to prove the following claim: if n m 2, then the
m2
rightmost m digits of 5n and 5n+2
agree in parity, but the (m + 1)-th digit from the
right differ in parity.
m2
(mod pk ).
n
X
cj (bk + pk r)j
j=0
k
X
k
0
cj (bjk + jpk rbj1
k ) = f (bk ) + p rf (bk )
(mod pk+1 ),
j=0
where the modulo equivalence comes from binomial expansion. (This is related to the
taylor expansion in calculus: f (x + ) f (x) + f 0 (x).) From the induction hypothesis,
we know pk | f (bk ). Also bk a (mod p), so p - f 0 (bk ), and hence f 0 (bk ) has an inverse
mod p, say t Z, satisfying f 0 (bk )t 1 (mod p). Then setting r = f (bpkk )t , we have
f (bk+1 ) f (bk ) f (bk )rf 0 (bk ) = f (bk )(1 rf 0 (bk )) 0
(mod pk+1 ).